Exploring Solutions to Spin Evolution Equations

In summary, a solution to the Schrodinger equation must take the form ##\begin{pmatrix} \Psi_1 \\ \Psi_2 \end{pmatrix}## and the energy operator will be as per usual ##\hat{E} = i\hbar \frac{\partial}{\partial t}##. I am confused by the fact that ##S_y = \frac{\hbar}{2} \begin{pmatrix} 0 & -i \\ i & 0 \end{pmatrix}## because the Schrodinger equation implies##\begin{pmatrix}\frac{\partial \Psi_1}{\partial t}\\\frac
  • #1
uxioq99
11
4
Homework Statement
Determine a wave function as a column vector of two states that solves the Schrodinger equation given by the Hamiltonian ##H=-\omega S_y##.
Relevant Equations
##H=-\omega S_y##
By the statement of the question, a solution must take the form
##\begin{pmatrix} \Psi_1 \\ \Psi_2 \end{pmatrix}##
and the energy operator will be as per usual ##\hat{E} = i\hbar \frac{\partial}{\partial t}##. I am confused by the fact that
##S_y = \frac{\hbar}{2} \begin{pmatrix} 0 & -i \\ i & 0 \end{pmatrix}##
because the Schrodinger equation implies
##\begin{pmatrix}\frac{\partial \Psi_1}{\partial t}\\\frac{\partial \Psi_2}{\partial t}\end{pmatrix}=\frac{\omega}{2}\begin{pmatrix}0 & -1 \\ 1 & 0\end{pmatrix}
\begin{pmatrix}
\Psi_1\\
\Psi_2
\end{pmatrix}
##

So that exponential solutions as opposed to oscillatory solutions are observed.

##
\begin{pmatrix}
\Psi_1(x) \\
\Psi_2(x)
\end{pmatrix}
=
\frac{1}{\sqrt{e^{\omega/2 t} + e^{-\omega/2 t}}}
\begin{pmatrix}
e^{-\omega/2 t} \\
e^{\omega/2 t}
\end{pmatrix}
##

Is this really a physically possible solution? Is it akin to when the energy of a bound state in a finite square well is below the depth of the well leading to an exponential decay in the solution? Now the decay appears to be in time, suggesting that the final state of the system will simply be ##\Psi_1 = 0## and ##\Psi_2 = 1##.

In general, if ##A \in \text{GL}_2(\mathbb R)## and

##
i\hbar\begin{pmatrix}
\frac{\partial \Psi_1}{\partial t}\\
\frac{\partial \Psi_2}{\partial t}
\end{pmatrix}=
i\hbar
\begin{pmatrix}a & b \\ c & d\end{pmatrix}
\begin{pmatrix}
\Psi_1\\
\Psi_2
\end{pmatrix}
##

Could one perform an eigendecomposition to exponentiate the spectrum of the matrix? The corresponding characteristic polynomial will be

##\lambda^2 - \lambda(a + d) + |A| = 0##

yielding

##\lambda_\pm = \frac{a+d}{2} \pm \sqrt{\frac{a+d}{2}^2 - |A|}##

The corresponding eigenvector matrix ##S## is too tedious to include here, but can be readily solved from the expression for the eigenvalues. Does that imply then, that the general solution is

##\begin{pmatrix}\Psi_1 \\ \Psi_2 \end{pmatrix} = S e^{\Lambda t} S^{-1}##

where ##\Lambda## is the diagonal matrix formed from ##\lambda_+## and ##\lambda_-##?

For either of these evolution equations, how would somebody specify the initial conditions so that the spin is taken along some vector in ##\mathbb{R}^3##?
 
Last edited:
Physics news on Phys.org
  • #2
uxioq99 said:
Homework Statement:: Determine a wave function as a column vector of two states that solves the Schrodinger equation given by the Hamiltonian ##H=-\omega S_y##.
Relevant Equations:: ##H=-\omega S_y##

By the statement of the question, a solution must take the form
##\begin{pmatrix} \Psi_1 \\ \Psi_2 \end{pmatrix}##
and the energy operator will be as per usual ##\hat{E} = i\hbar \frac{\partial}{\partial t}##. I am confused by the fact that
##S_y = \frac{\hbar}{2} \begin{pmatrix} 0 & -i \\ i & 0 \end{pmatrix}##
because the Schrodinger equation implies
##\begin{pmatrix}\frac{\partial \Psi_1}{\partial t}\\\frac{\partial \Psi_2}{\partial t}\end{pmatrix}=\frac{\omega}{2}\begin{pmatrix}0 & -1 \\ 1 & 0\end{pmatrix}##

So that exponential solutions as opposed to oscillatory solutions are observed.

##
\begin{pmatrix}
\Psi_1(x) \\
\Psi_2(x)
\end{pmatrix}
=
\frac{1}{\sqrt{e^{\omega/2 t} + e^{-\omega/2 t}}}
\begin{pmatrix}
e^{-\omega/2 t} \\
e^{\omega/2 t}
\end{pmatrix}
##

Is this really a physically possible solution? Is it akin to when the energy of a bound state in a finite square well is below the depth of the well leading to an exponential decay in the solution? Now the decay appears to be in time, suggesting that the final state of the system will simply be ##\Psi_1 = 0## and ##\Psi_2 = 1##.

In general, if ##A \in \text{GL}_2(\mathbb R)## and

##
i\hbar\begin{pmatrix}
\frac{\partial \Psi_1}{\partial t}\\
\frac{\partial \Psi_2}{\partial t}
\end{pmatrix}=
i\hbar
\begin{pmatrix}a & b \\ c & d\end{pmatrix}
##

Could one perform an eigendecomposition to exponentiate the spectrum of the matrix? The corresponding characteristic polynomial will be

##\lambda^2 - \lambda(a + d) + |A| = 0##

yielding

##\lambda_\pm = \frac{a+d}{2} \pm \sqrt{\frac{a+d}{2}^2 - |A|}##

The corresponding eigenvector matrix ##S## is too tedious to include here, but can be readily solved from the expression for the eigenvalues. Does that imply then, that the general solution is

##\begin{pmatrix}\Psi_1 \\ \Psi_2 \end{pmatrix} = S e^{\Lambda t} S^{-1}##

where ##\Lambda## is the diagonal matrix formed from ##\lambda_+## and ##\lambda_-##?

For either of these evolution equations, how would somebody specify the initial conditions so that the spin is taken along some vector in ##\mathbb{R}^3##?
##\begin{pmatrix}\frac{\partial \Psi_1}{\partial t}\\\frac{\partial \Psi_2}{\partial t}\end{pmatrix}=\frac{\omega}{2}\begin{pmatrix}0 & -1 \\ 1 & 0\end{pmatrix}##
What happened to the ##\Psi## column vector on the Right Hand Side?
 
  • Like
Likes topsquark and uxioq99
  • #3
Thanks, I added it back into the equation, right at the end of the editing window.
 
  • #4
The rest is just incorrect. The solution you write down is not a solution. The exponent is not dimensionless and then it won't work anyway . At that point I stopped.
Get a copy of any good quantum text and work it through. Give it another attempt.
It is not nonsense......just full of mistakes Might be good to number your equations.
 
  • Like
Likes topsquark
  • #5
@hutchphd Sorry, I realized when I typed "\exp(-\omega/2t)" it produced ##\exp(-\omega/2t)## instead of ##\exp(-\frac{\omega}{2} t)## Wouldn't the units of ##\omega## cancel those of time given that it is an angular frequency? Would

##
\begin{pmatrix}
\Psi_1(x,t) \\
\Psi_2(x,t)
\end{pmatrix}
=
\begin{pmatrix}
\exp(\frac{\omega}{2} t) \\
\exp(-\frac{\omega}{2} t)
\end{pmatrix}
##

Be a valid solution of ##E\vec\Psi = H \vec \Psi## where ##H = -\omega S_y##?

Wouldn't

##
-i\hbar
\begin{pmatrix}
\frac{\partial \Psi_1}{\partial t} \\
\frac{\partial \Psi_2}{\partial t}
\end{pmatrix} =
-\frac{\hbar \omega}{2} \begin{pmatrix}
0 & -i \\
i & 0
\end{pmatrix}
\begin{pmatrix}
\Psi_1 (x,t) \\
\Psi_2 (x,t)
\end{pmatrix}
##

because

##
-i\hbar \frac{\partial}{\partial t} \left(e^{\pm\frac{\omega}{2} t}\right) =
\mp i \frac{\hbar \omega}{2} e^{\pm\frac{\omega}{2} t}
##

and

##
-\frac{\hbar \omega}{2}
\begin{pmatrix}
0 & -i \\
i & 0
\end{pmatrix}
\begin{pmatrix}
e^{-\frac{\omega}{2} t} \\
e^{\frac{\omega}{2} t}
\end{pmatrix} =
i\frac{\hbar \omega}{2}
\begin{pmatrix}
-e^{\frac{\omega}{2} t} \\
e^{-\frac{\omega}{2} t}
\end{pmatrix}
##

I believe that a solution to the Schrodinger equation cannot always be real, so my selected form for ##\Psi(x,t)## does not appear to be correct. Additionally, the normalization factor I mentioned earlier

##\frac{1}{\sqrt{e^{\omega t} + e^{-\omega t}}}##

is time dependent. I know that I neglected the appropriate derivatives in the energy operator, but I was unsure of what else to do.

When I consider the similar problem for ##H = -\omega S_z##, I had that the Hamiltonian became

##
H = -\frac{\hbar \omega}{2} \begin{pmatrix}
1 & 0 \\
0 & -1
\end{pmatrix}
##

that admitted a solution

##
\begin{pmatrix}
\Psi_1(x,t) \\
\Psi_2(x,t)
\end{pmatrix} =
\frac{1}{\sqrt{2}}\begin{pmatrix}
e^{i\frac{\omega}{2} t} \\
e^{-i\frac{\omega}{2} t}
\end{pmatrix}
##

The normalization ##\frac{1}{\sqrt{2}}## remained constant in time because of the presence of these oscillators. I am confused by the presence of the imaginary unit ##i## in the ##S_y##. It must be present by definition, but it cancels the ##i## in the energy operator ##E = i \hbar \frac{\partial}{\partial t}##. Then, the solutions appear to be exponential instead of oscillatory. This fact conflicted my intuition because -- as said previously -- the wavefunction is real and does not appear to be easily normalizable.

Also regarding my generalization to any 2 by 2 matrix. I am aware that the Pauli matrices span ##\mathbb{C}^{2\times 2}## and so the general solution could be written by decomposing the Hamiltonian into these constituent matrices. I had hoped that an alternate characterization in purely the language of introductory linear algebra could be fashioned by examining the equation as a system of ODEs and considering the spectrum. If
##
\frac{d\vec x}{dt} = A \vec x(t)
##
then
##
\vec x (t) = S\exp(\Lambda t) S^{-1} \vec x_0
##
by considering ##\exp## as a matrix power series.
 
Last edited:
  • #6
uxioq99 said:
the Schrodinger equation implies
##\begin{pmatrix}\frac{\partial \Psi_1}{\partial t}\\\frac{\partial \Psi_2}{\partial t}\end{pmatrix}=\frac{\omega}{2}\begin{pmatrix}0 & -1 \\ 1 & 0\end{pmatrix}
\begin{pmatrix}
\Psi_1\\
\Psi_2
\end{pmatrix}
##
On the right side, check to see if you missed an overall minus sign.

If you multiply out the right hand side, what column vector do you get?
 
  • Like
Likes uxioq99
  • #7
@TSny Thank you, dividing by ##i## introduces the negative sign. I make more mistakes when I type things in Latex. Is the solution still supposed to be exponential, or should I have instead looked for a oscillatory behavior? I don't see how that is possible, given that the matrix defining the system becomes real. (Unless, I forgot an ##i## in addition to a negative sign.) My solution appears wrong because it no longer carries a phase.
 
  • #8
You shouldn't get exponentially growing or damping solutions.

What do you get after performing the matrix multiplication on the right-hand side of the equation below?

##\begin{pmatrix}\frac{\partial \Psi_1}{\partial t}\\\frac{\partial \Psi_2}{\partial t}\end{pmatrix}=\frac{\omega}{2}\begin{pmatrix}0 & 1 \\ -1 & 0\end{pmatrix}\begin{pmatrix}\Psi_1\\\Psi_2\end{pmatrix}##
 
  • Like
Likes hutchphd
  • #9
I'm quite embarrassed. The values are on the off-diagonal. I apologize for asking this in the first place. I've been studying too long and now am just making stupid mistakes. Even by my standards, this was an incredible mistake -- sorry.
 
  • Like
Likes hutchphd
  • #10
No problem. I’m glad it’s resolved.
 
  • Like
Likes hutchphd

1. What are spin evolution equations?

Spin evolution equations are mathematical equations that describe how the spin of a particle changes over time. They are used in quantum mechanics to understand the behavior of particles with spin, such as electrons.

2. Why is it important to explore solutions to spin evolution equations?

Exploring solutions to spin evolution equations helps us better understand the behavior of particles with spin and can lead to advancements in fields such as quantum computing and materials science.

3. How do scientists approach solving spin evolution equations?

Scientists use a variety of mathematical techniques, such as perturbation theory and numerical methods, to solve spin evolution equations. They also use experimental data to validate their solutions.

4. What are some real-world applications of spin evolution equations?

Spin evolution equations have applications in fields such as quantum computing, magnetic resonance imaging (MRI), and spintronics. They are also used in understanding the behavior of particles in accelerators and in studying the properties of materials.

5. Are there any current challenges in exploring solutions to spin evolution equations?

Yes, there are still many challenges in solving spin evolution equations, such as the need for more accurate and efficient numerical methods and the difficulty in incorporating interactions between particles. Additionally, there is ongoing research to understand the behavior of particles with spin in extreme conditions, such as high energies or strong magnetic fields.

Similar threads

Replies
3
Views
855
  • Advanced Physics Homework Help
Replies
8
Views
729
  • Advanced Physics Homework Help
Replies
3
Views
352
  • Advanced Physics Homework Help
Replies
13
Views
1K
  • Advanced Physics Homework Help
Replies
3
Views
1K
  • Advanced Physics Homework Help
Replies
14
Views
1K
  • Advanced Physics Homework Help
Replies
3
Views
1K
  • Advanced Physics Homework Help
Replies
7
Views
753
  • Advanced Physics Homework Help
Replies
1
Views
710
  • Advanced Physics Homework Help
Replies
13
Views
1K
Back
Top